A 40-year-old man presents to the Emergency Department after fainting while exercising. He reports past episodes of chest pain, which have occurred both at rest and with exercise.... A 40-year-old man presents to the Emergency Department after fainting while exercising. He reports past episodes of chest pain, which have occurred both at rest and with exercise. On examination of the chest, there is a systolic ejection murmur heard at the left sternal border, which does not radiate. The intensity of the murmur increases when the patient bears down or stands up. The lung fields are clear. Blood pressure 125/75 mmHg, Heart rate 110/min, Respiratory rate 18/min, Temperature 36.6°C, ECG shows non-specific ST and T wave changes with left atrial enlargement. Which of the following is the most likely diagnosis? A. Aortic stenosis B. Pulmonic stenosis.

Question image

Understand the Problem

The question presents a medical case scenario about a 40-year-old man with specific symptoms and asks for the most likely diagnosis based on the provided clinical information. It prompts the reader to analyze the details of the symptoms, vital signs, and ECG findings to determine the appropriate medical condition.

Answer

Hypertrophic cardiomyopathy

The final answer is hypertrophic cardiomyopathy.

Answer for screen readers

The final answer is hypertrophic cardiomyopathy.

More Information

Hypertrophic cardiomyopathy often presents with a systolic murmur that increases with maneuvers like standing up or bearing down due to decreased preload, consistent with the murmur's behavior described.

Tips

Misidentifying the murmur dynamics as aortic stenosis, which typically doesn't increase with standing.

AI-generated content may contain errors. Please verify critical information

Thank you for voting!
Use Quizgecko on...
Browser
Browser